Merge remote branch 'public/master'
[course.git] / latex / problems / Serway_and_Jewett_4 / problem10.44.tex
1 \begin{problem*}{10.44}
2 A space station is constructed in the shape of a hollow ring of mass
3 $m = 5.00\E{4}\U{kg}$.  Members of the crew walk on a deck formed by
4 the inner surface of the outer cylindrical wall of the ring, with a
5 radius of $r = 100\U{m}$.  At rest when constructed, the ring is set
6 rotating about its axis so that the people inside experience an
7 effective free-fall acceleration equal to $g$.  The rotation is
8 achieved by firing two small rockets attached tangentially at opposite
9 points on the outside of the ring.
10 \Part{a} What angular momentum does the space station acquire?
11 \Part{b} How long must the rockets be fired if each exerts a thrust
12 of $F = 125\U{N}$?
13 \Part{c} Prove that the total torque on the ring, multiplied by the
14 the time interval found in \Part{b}, is equal to the change in angular
15 momentum found in \Part{a}.  This equality represents the
16 \emph{angular impulse-angular momentum theorem}.
17 \end{problem*} % problem 10.44
18
19 \begin{solution}
20 \Part{a}
21 The certerward acceleration of people on the wall of the space station
22 is given by
23 \begin{align}
24  g = a_c &= \frac{v^2}{r} = r \omega^2 \\
25  \omega &= \sqrt{\frac{g}{r}}
26 \end{align}
27 Where we used $v = r\omega$ to replace the linear velocity $v$.  The
28 moment of inertia of a ring is given by $I = mr^2$ from table 10.2 on
29 page 300.  The angular momentum is then given by
30 \begin{equation}
31  L = I \omega = m r^2 \sqrt{\frac{g}{r}} = \ans{m r \sqrt{gr}} = \ans{1.57\E{8}\U{Js}}
32 \end{equation}
33
34 \Part{b}
35 The torque on the station is given by
36 \begin{align}
37  \sum \tau &= 2 \cdot r \cdot F = I \alpha = m r^2 \alpha\\
38   \alpha &= \frac{2 F}{m r}
39 \end{align}
40 Going back to our constant acceleration equations, we see that
41 \begin{align}
42  \omega &= \alpha t + \omega_0 = \alpha t \\
43   t &= \frac{\omega}{\alpha} = \sqrt{\frac{g}{r}} \cdot \frac{m r}{2 F} = \sqrt{g r} \frac{m}{2F}
44      = \sqrt{ 9.80\U{m/s}^2 \cdot 100\U{m}} \frac{5\E{4}\U{kg}}{2 \cdot 125\U{N}}
45      = \ans{ 6.26\U{ks} = 1.74\U{hr} }
46 \end{align}
47
48 \Part{c}
49 \begin{align}
50  \tau t &= I \alpha t = I \omega = L \\
51  2 r F t &= 2 \cdot 100\U{m} \cdot 125\U{N} \cdot 6.26\E{3}\U{s}
52           = 1.57\E{8}\U{Js} = L
53 \end{align}
54 So they are equal both symbolically and numerically which means I
55 probably didn't make any algebra mistakes (we can hope).
56 \end{solution}